Namespaces
Variants
Actions

Difference between revisions of "Congruence with several variables"

From Encyclopedia of Mathematics
Jump to: navigation, search
(Importing text file)
 
m (tex encoded by computer)
 
Line 1: Line 1:
 +
<!--
 +
c0249401.png
 +
$#A+1 = 40 n = 0
 +
$#C+1 = 40 : ~/encyclopedia/old_files/data/C024/C.0204940 Congruence with several variables
 +
Automatically converted into TeX, above some diagnostics.
 +
Please remove this comment and the {{TEX|auto}} line below,
 +
if TeX found to be correct.
 +
-->
 +
 +
{{TEX|auto}}
 +
{{TEX|done}}
 +
 
A congruence
 
A congruence
  
<table class="eq" style="width:100%;"> <tr><td valign="top" style="width:94%;text-align:center;"><img align="absmiddle" border="0" src="https://www.encyclopediaofmath.org/legacyimages/c/c024/c024940/c0249401.png" /></td> <td valign="top" style="width:5%;text-align:right;">(1)</td></tr></table>
+
$$ \tag{1 }
 +
f( x _ {1} \dots x _ {n} )  \equiv  0 ( \mathop{\rm mod}  m),
 +
$$
  
where <img align="absmiddle" border="0" src="https://www.encyclopediaofmath.org/legacyimages/c/c024/c024940/c0249402.png" /> is a polynomial in <img align="absmiddle" border="0" src="https://www.encyclopediaofmath.org/legacyimages/c/c024/c024940/c0249403.png" /> variables with integer rational coefficients not all of which are divisible by <img align="absmiddle" border="0" src="https://www.encyclopediaofmath.org/legacyimages/c/c024/c024940/c0249404.png" />. The solvability of this congruence modulo <img align="absmiddle" border="0" src="https://www.encyclopediaofmath.org/legacyimages/c/c024/c024940/c0249405.png" />, where <img align="absmiddle" border="0" src="https://www.encyclopediaofmath.org/legacyimages/c/c024/c024940/c0249406.png" /> are different prime numbers, is equivalent to the solvability of the congruences
+
where $  f( x _ {1} \dots x _ {n} ) $
 +
is a polynomial in $  n \geq  2 $
 +
variables with integer rational coefficients not all of which are divisible by $  m $.  
 +
The solvability of this congruence modulo $  m = p _ {1} ^ {\alpha _ {1} } {} \dots p _ {s} ^ {\alpha _ {s} } $,  
 +
where $  p _ {1} \dots p _ {s} $
 +
are different prime numbers, is equivalent to the solvability of the congruences
  
<table class="eq" style="width:100%;"> <tr><td valign="top" style="width:94%;text-align:center;"><img align="absmiddle" border="0" src="https://www.encyclopediaofmath.org/legacyimages/c/c024/c024940/c0249407.png" /></td> <td valign="top" style="width:5%;text-align:right;">(2)</td></tr></table>
+
$$ \tag{2 }
 +
f( x _ {1} \dots x _ {n} )  \equiv \
 +
0 ( \mathop{\rm mod}  p _ {i} ^ {\alpha _ {i} } )
 +
$$
  
for all <img align="absmiddle" border="0" src="https://www.encyclopediaofmath.org/legacyimages/c/c024/c024940/c0249408.png" />. The number <img align="absmiddle" border="0" src="https://www.encyclopediaofmath.org/legacyimages/c/c024/c024940/c0249409.png" /> of solutions of (1) is then equal to the product <img align="absmiddle" border="0" src="https://www.encyclopediaofmath.org/legacyimages/c/c024/c024940/c02494010.png" />, where <img align="absmiddle" border="0" src="https://www.encyclopediaofmath.org/legacyimages/c/c024/c024940/c02494011.png" /> is the number of solutions of (2). Thus, when studying congruences of the form (1) it is sufficient to confine oneself to moduli that are powers of prime numbers.
+
for all $  i = 1 \dots s $.  
 +
The number $  N $
 +
of solutions of (1) is then equal to the product $  N _ {1} \dots N _ {s} $,  
 +
where $  N _ {i} $
 +
is the number of solutions of (2). Thus, when studying congruences of the form (1) it is sufficient to confine oneself to moduli that are powers of prime numbers.
  
 
For a congruence
 
For a congruence
  
<table class="eq" style="width:100%;"> <tr><td valign="top" style="width:94%;text-align:center;"><img align="absmiddle" border="0" src="https://www.encyclopediaofmath.org/legacyimages/c/c024/c024940/c02494012.png" /></td> <td valign="top" style="width:5%;text-align:right;">(3)</td></tr></table>
+
$$ \tag{3 }
 +
f( x _ {1} \dots x _ {n} )  \equiv  0 ( \mathop{\rm mod}  p  ^  \alpha  ),\ \
 +
\alpha > 1,
 +
$$
  
 
to be solvable, it is necessary that the congruence
 
to be solvable, it is necessary that the congruence
  
<table class="eq" style="width:100%;"> <tr><td valign="top" style="width:94%;text-align:center;"><img align="absmiddle" border="0" src="https://www.encyclopediaofmath.org/legacyimages/c/c024/c024940/c02494013.png" /></td> <td valign="top" style="width:5%;text-align:right;">(4)</td></tr></table>
+
$$ \tag{4 }
 +
f( x _ {1} \dots x _ {n} )  \equiv  0 ( \mathop{\rm mod}  p)
 +
$$
  
modulo a prime number <img align="absmiddle" border="0" src="https://www.encyclopediaofmath.org/legacyimages/c/c024/c024940/c02494014.png" /> be solvable. In non-degenerate cases, the solvability of (4) is also a sufficient condition for the solvability of (3). More precisely, the following statement is correct: Every solution <img align="absmiddle" border="0" src="https://www.encyclopediaofmath.org/legacyimages/c/c024/c024940/c02494015.png" /> (<img align="absmiddle" border="0" src="https://www.encyclopediaofmath.org/legacyimages/c/c024/c024940/c02494016.png" />) of (4) such that <img align="absmiddle" border="0" src="https://www.encyclopediaofmath.org/legacyimages/c/c024/c024940/c02494017.png" /> (<img align="absmiddle" border="0" src="https://www.encyclopediaofmath.org/legacyimages/c/c024/c024940/c02494018.png" />) for at least one <img align="absmiddle" border="0" src="https://www.encyclopediaofmath.org/legacyimages/c/c024/c024940/c02494019.png" />, generates <img align="absmiddle" border="0" src="https://www.encyclopediaofmath.org/legacyimages/c/c024/c024940/c02494020.png" /> solutions <img align="absmiddle" border="0" src="https://www.encyclopediaofmath.org/legacyimages/c/c024/c024940/c02494021.png" /> (<img align="absmiddle" border="0" src="https://www.encyclopediaofmath.org/legacyimages/c/c024/c024940/c02494022.png" />) of (3), whereby <img align="absmiddle" border="0" src="https://www.encyclopediaofmath.org/legacyimages/c/c024/c024940/c02494023.png" /> (<img align="absmiddle" border="0" src="https://www.encyclopediaofmath.org/legacyimages/c/c024/c024940/c02494024.png" />) when <img align="absmiddle" border="0" src="https://www.encyclopediaofmath.org/legacyimages/c/c024/c024940/c02494025.png" />.
+
modulo a prime number $  p $
 +
be solvable. In non-degenerate cases, the solvability of (4) is also a sufficient condition for the solvability of (3). More precisely, the following statement is correct: Every solution $  x _ {i} \equiv x _ {i}  ^ {(} 1) $(
 +
$  \mathop{\rm mod}  p $)  
 +
of (4) such that $  ( df/dx _ {i} )( x _ {1}  ^ {(} 1) \dots x _ {n}  ^ {(} 1) ) \not\equiv 0 $(
 +
$  \mathop{\rm mod}  p $)  
 +
for at least one $  i = 1 \dots n $,  
 +
generates $  p ^ {( \alpha - 1)( n- 1) } $
 +
solutions $  x _ {i} \equiv x _ {i} ^ {( \alpha ) } $(
 +
$  \mathop{\rm mod}  p  ^  \alpha  $)  
 +
of (3), whereby $  x _ {i} ^ {( \alpha ) } \equiv x _ {i}  ^ {(} 1) $(
 +
$  \mathop{\rm mod}  p $)  
 +
when $  i = 1 \dots n $.
  
Thus, in the non-degenerate case, the question of the number of solutions of the congruence (1) modulo a composite number <img align="absmiddle" border="0" src="https://www.encyclopediaofmath.org/legacyimages/c/c024/c024940/c02494026.png" /> reduces to the question of the number of solutions of congruences of the form (4) modulo the prime numbers <img align="absmiddle" border="0" src="https://www.encyclopediaofmath.org/legacyimages/c/c024/c024940/c02494027.png" /> that divide <img align="absmiddle" border="0" src="https://www.encyclopediaofmath.org/legacyimages/c/c024/c024940/c02494028.png" />. If <img align="absmiddle" border="0" src="https://www.encyclopediaofmath.org/legacyimages/c/c024/c024940/c02494029.png" /> is an absolutely-irreducible polynomial with integer rational coefficients, then for the number <img align="absmiddle" border="0" src="https://www.encyclopediaofmath.org/legacyimages/c/c024/c024940/c02494030.png" /> of solutions of (4), the estimate
+
Thus, in the non-degenerate case, the question of the number of solutions of the congruence (1) modulo a composite number $  m $
 +
reduces to the question of the number of solutions of congruences of the form (4) modulo the prime numbers $  p $
 +
that divide $  m $.  
 +
If $  f( x _ {1} \dots x _ {n} ) $
 +
is an absolutely-irreducible polynomial with integer rational coefficients, then for the number $  N _ {p} $
 +
of solutions of (4), the estimate
  
<table class="eq" style="width:100%;"> <tr><td valign="top" style="width:94%;text-align:center;"><img align="absmiddle" border="0" src="https://www.encyclopediaofmath.org/legacyimages/c/c024/c024940/c02494031.png" /></td> </tr></table>
+
$$
 +
| N _ {p} - p  ^ {n-} 1 |  \leq  C( f  ) p  ^ {n-} 1- 1/2
 +
$$
  
holds, where the constant <img align="absmiddle" border="0" src="https://www.encyclopediaofmath.org/legacyimages/c/c024/c024940/c02494032.png" /> depends only on <img align="absmiddle" border="0" src="https://www.encyclopediaofmath.org/legacyimages/c/c024/c024940/c02494033.png" /> and does not depend on <img align="absmiddle" border="0" src="https://www.encyclopediaofmath.org/legacyimages/c/c024/c024940/c02494034.png" />. It follows from this estimate that the congruence (4) is solvable for all prime numbers <img align="absmiddle" border="0" src="https://www.encyclopediaofmath.org/legacyimages/c/c024/c024940/c02494035.png" /> that are larger than a certain effectively-calculable constant <img align="absmiddle" border="0" src="https://www.encyclopediaofmath.org/legacyimages/c/c024/c024940/c02494036.png" />, depending on the given polynomial <img align="absmiddle" border="0" src="https://www.encyclopediaofmath.org/legacyimages/c/c024/c024940/c02494037.png" /> (see also [[Congruence modulo a prime number|Congruence modulo a prime number]]). A stronger result in this question has been obtained by P. Deligne [[#References|[3]]].
+
holds, where the constant $  C( f  ) $
 +
depends only on $  f $
 +
and does not depend on $  p $.  
 +
It follows from this estimate that the congruence (4) is solvable for all prime numbers $  p $
 +
that are larger than a certain effectively-calculable constant $  C _ {0} ( f  ) $,  
 +
depending on the given polynomial $  f( x _ {1} \dots x _ {n} ) $(
 +
see also [[Congruence modulo a prime number|Congruence modulo a prime number]]). A stronger result in this question has been obtained by P. Deligne [[#References|[3]]].
  
 
====References====
 
====References====
 
<table><TR><TD valign="top">[1]</TD> <TD valign="top">  Z.I. Borevich,  I.R. Shafarevich,  "Number theory" , Acad. Press  (1966)  (Translated from Russian)  (German translation: Birkhäuser, 1966)</TD></TR><TR><TD valign="top">[2]</TD> <TD valign="top">  H. Hasse,  "Zahlentheorie" , Akademie Verlag  (1963)</TD></TR><TR><TD valign="top">[3]</TD> <TD valign="top">  P. Deligne,  "La conjecture de Weil I"  ''Publ. Math. IHES'' , '''43'''  (1974)  pp. 273–307</TD></TR></table>
 
<table><TR><TD valign="top">[1]</TD> <TD valign="top">  Z.I. Borevich,  I.R. Shafarevich,  "Number theory" , Acad. Press  (1966)  (Translated from Russian)  (German translation: Birkhäuser, 1966)</TD></TR><TR><TD valign="top">[2]</TD> <TD valign="top">  H. Hasse,  "Zahlentheorie" , Akademie Verlag  (1963)</TD></TR><TR><TD valign="top">[3]</TD> <TD valign="top">  P. Deligne,  "La conjecture de Weil I"  ''Publ. Math. IHES'' , '''43'''  (1974)  pp. 273–307</TD></TR></table>
 
 
  
 
====Comments====
 
====Comments====
See also [[Congruence equation|Congruence equation]] for more information. A polynomial <img align="absmiddle" border="0" src="https://www.encyclopediaofmath.org/legacyimages/c/c024/c024940/c02494038.png" /> over <img align="absmiddle" border="0" src="https://www.encyclopediaofmath.org/legacyimages/c/c024/c024940/c02494039.png" /> is absolutely irreducible if it is still irreducible over any (algebraic) field extension of <img align="absmiddle" border="0" src="https://www.encyclopediaofmath.org/legacyimages/c/c024/c024940/c02494040.png" />.
+
See also [[Congruence equation|Congruence equation]] for more information. A polynomial $  f ( x _ {1} \dots x _ {n} ) $
 +
over $  \mathbf Q $
 +
is absolutely irreducible if it is still irreducible over any (algebraic) field extension of $  \mathbf Q $.

Latest revision as of 17:46, 4 June 2020


A congruence

$$ \tag{1 } f( x _ {1} \dots x _ {n} ) \equiv 0 ( \mathop{\rm mod} m), $$

where $ f( x _ {1} \dots x _ {n} ) $ is a polynomial in $ n \geq 2 $ variables with integer rational coefficients not all of which are divisible by $ m $. The solvability of this congruence modulo $ m = p _ {1} ^ {\alpha _ {1} } {} \dots p _ {s} ^ {\alpha _ {s} } $, where $ p _ {1} \dots p _ {s} $ are different prime numbers, is equivalent to the solvability of the congruences

$$ \tag{2 } f( x _ {1} \dots x _ {n} ) \equiv \ 0 ( \mathop{\rm mod} p _ {i} ^ {\alpha _ {i} } ) $$

for all $ i = 1 \dots s $. The number $ N $ of solutions of (1) is then equal to the product $ N _ {1} \dots N _ {s} $, where $ N _ {i} $ is the number of solutions of (2). Thus, when studying congruences of the form (1) it is sufficient to confine oneself to moduli that are powers of prime numbers.

For a congruence

$$ \tag{3 } f( x _ {1} \dots x _ {n} ) \equiv 0 ( \mathop{\rm mod} p ^ \alpha ),\ \ \alpha > 1, $$

to be solvable, it is necessary that the congruence

$$ \tag{4 } f( x _ {1} \dots x _ {n} ) \equiv 0 ( \mathop{\rm mod} p) $$

modulo a prime number $ p $ be solvable. In non-degenerate cases, the solvability of (4) is also a sufficient condition for the solvability of (3). More precisely, the following statement is correct: Every solution $ x _ {i} \equiv x _ {i} ^ {(} 1) $( $ \mathop{\rm mod} p $) of (4) such that $ ( df/dx _ {i} )( x _ {1} ^ {(} 1) \dots x _ {n} ^ {(} 1) ) \not\equiv 0 $( $ \mathop{\rm mod} p $) for at least one $ i = 1 \dots n $, generates $ p ^ {( \alpha - 1)( n- 1) } $ solutions $ x _ {i} \equiv x _ {i} ^ {( \alpha ) } $( $ \mathop{\rm mod} p ^ \alpha $) of (3), whereby $ x _ {i} ^ {( \alpha ) } \equiv x _ {i} ^ {(} 1) $( $ \mathop{\rm mod} p $) when $ i = 1 \dots n $.

Thus, in the non-degenerate case, the question of the number of solutions of the congruence (1) modulo a composite number $ m $ reduces to the question of the number of solutions of congruences of the form (4) modulo the prime numbers $ p $ that divide $ m $. If $ f( x _ {1} \dots x _ {n} ) $ is an absolutely-irreducible polynomial with integer rational coefficients, then for the number $ N _ {p} $ of solutions of (4), the estimate

$$ | N _ {p} - p ^ {n-} 1 | \leq C( f ) p ^ {n-} 1- 1/2 $$

holds, where the constant $ C( f ) $ depends only on $ f $ and does not depend on $ p $. It follows from this estimate that the congruence (4) is solvable for all prime numbers $ p $ that are larger than a certain effectively-calculable constant $ C _ {0} ( f ) $, depending on the given polynomial $ f( x _ {1} \dots x _ {n} ) $( see also Congruence modulo a prime number). A stronger result in this question has been obtained by P. Deligne [3].

References

[1] Z.I. Borevich, I.R. Shafarevich, "Number theory" , Acad. Press (1966) (Translated from Russian) (German translation: Birkhäuser, 1966)
[2] H. Hasse, "Zahlentheorie" , Akademie Verlag (1963)
[3] P. Deligne, "La conjecture de Weil I" Publ. Math. IHES , 43 (1974) pp. 273–307

Comments

See also Congruence equation for more information. A polynomial $ f ( x _ {1} \dots x _ {n} ) $ over $ \mathbf Q $ is absolutely irreducible if it is still irreducible over any (algebraic) field extension of $ \mathbf Q $.

How to Cite This Entry:
Congruence with several variables. Encyclopedia of Mathematics. URL: http://encyclopediaofmath.org/index.php?title=Congruence_with_several_variables&oldid=18763
This article was adapted from an original article by S.A. Stepanov (originator), which appeared in Encyclopedia of Mathematics - ISBN 1402006098. See original article